Q10

 
cdjmarmon
Thanks Received: 0
Elle Woods
Elle Woods
 
Posts: 59
Joined: July 12th, 2011
 
 
 

Q10

by cdjmarmon Mon Feb 06, 2012 8:29 pm

Im confused about question 10 in regards to the answers. Mostly the last two. They confuse me becuase they do not indicate if there isonly 5 or only 6 spaces.

For instance, if there is only 5 spaces then L couldnt go in the 5th space. But if there is 6 spaces then L could. Like wise for if there is only 4 spaces L couldnt go in the 4th space but if there is 5 or 6 spaces then L could.
 
timmydoeslsat
Thanks Received: 887
Atticus Finch
Atticus Finch
 
Posts: 1136
Joined: June 20th, 2011
 
 
trophy
Most Thanked
trophy
First Responder
 

Re: Q10

by timmydoeslsat Mon Feb 06, 2012 11:17 pm

cdjmarmon Wrote:Im confused about question 10 in regards to the answers. Mostly the last two. They confuse me becuase they do not indicate if there isonly 5 or only 6 spaces.

For instance, if there is only 5 spaces then L couldnt go in the 5th space. But if there is 6 spaces then L could. Like wise for if there is only 4 spaces L couldnt go in the 4th space but if there is 5 or 6 spaces then L could.


There are 3 people and each must have sent from 1-2 messages.

The lowest number of total messages we can have is 4. It was 3 initially before reading the rules, but with the first-last is same person rule, and L not going first, we can infer that there must be at least 4 messages.

If there were only 4 messages, then you are correct that L cannot go there, as that would imply L going first, which is not allowed.

However, these answer choices do not state out of how many total messages there are.

Without going into hypotheticals for each answer choice, I want to keep in mind that this question stem is a global what must be false question. I am banking on one of these answer choices sticking out to me.

A) I don't see why L could not go 2.
B) Ditto on 3
C) Ditto on 4

At this point I am not sure if I may have possibly skipped over the possible correct answer, but it is ok, I am being wise in first searching for a can't miss answer instead of running hypotheticals.

D) 5th, if there is a 5th. Well, for L to be 5th, there must be 6 total messages to make this happen, as L cannot be first.

So I can quickly run 2 hypos to make sure this checks out. I will either have H-H or J-J first last situation. I know I have to have 1 HJ block, so I know I cannot have a J-J as 1-6, because with L being 5th in this answer choice, this would stop our HJ block situation from occurring. So I know I am dealing with a H-H 1-6 option with L going 5th. I know that I have to have a J go 2nd, so I conform to the 1 HJ block rule.

H J _ _ L H

I know that I must have only one J in the first 3 messages. Since my H's are exhausted (max of 2 messages per person) I am required to place L 3rd, which leaves me to place J fourth. This works. This is not our answer because I validly placed L 5th, so this does not have to be a must be false situation.

E) L being 6 if there are 6. The maximum number of messages we can have is 6. If there are 6, L could never be 6! That would require L being first, which is a rule violation. This is our answer.
 
giladedelman
Thanks Received: 833
LSAT Geek
 
Posts: 619
Joined: April 04th, 2010
 
 
 

Re: Q10

by giladedelman Wed Feb 08, 2012 12:51 pm

Love this explanation! Especially this part:

"Without going into hypotheticals for each answer choice, I want to keep in mind that this question stem is a global what must be false question. I am banking on one of these answer choices sticking out to me.

A) I don't see why L could not go 2.
B) Ditto on 3
C) Ditto on 4

At this point I am not sure if I may have possibly skipped over the possible correct answer, but it is ok, I am being wise in first searching for a can't miss answer instead of running hypotheticals."


That's the right attitude! I would say you could even take it further and hold off until you see all the answer choices -- (E) is a pretty clear winner here. BUT, in the heat of battle, I totally see why (D) and (E) would both appear to be worth trying out.
 
giladedelman
Thanks Received: 833
LSAT Geek
 
Posts: 619
Joined: April 04th, 2010
 
 
 

Re: Q10

by giladedelman Wed Feb 08, 2012 1:07 pm

And just to respond more directly to the first poster: you're absolutely right that if there are 5 spaces, L couldn't go 5th, but if there are 6, L could. Since the question doesn't specify, all possibilities are in play, which means L could be 5th, so this doesn't have to be false.

On the other hand, L could never go 6th, so (E) must be false.

Does that answer your question?
 
giladedelman
Thanks Received: 833
LSAT Geek
 
Posts: 619
Joined: April 04th, 2010
 
 
 

Re: Q10

by giladedelman Fri Dec 14, 2012 3:26 pm

My "official" explanation for the question itself.

10. (E)
Question Type: Unconditional


This question asks us to identify a position where L cannot go. Sound familiar? It should: one of the constraints tells us that L can’t be first, and in our initial setup we inferred that this meant L can’t go last, either. Since "first" would be far too easy an answer, we can be fairly confident that the correct answer will depend on that inference about L going last.

Turning to the answer choices, we see that, indeed, every position is represented except first. We could go through each choice, to see whether L could fit there, but why bother? We know we’re looking for "last." If we skim to the bottom, we see that answer (E) is "the sixth message (if there is a sixth one)." Since six is the maximum, we know the sixth message has to be the last, so L can’t go there. Let’s circle (E) and move on!